Inscription / Connexion Nouveau Sujet
Niveau LicenceMaths 2e/3e a
Partager :

forme quadratique

Posté par
scoatarin
16-03-18 à 12:24

Bonjour,

Le mois dernier, je n'ai pas réussi à résoudre la question 6 de l'exercice suivant.

Comment faire ?  

Exercice 1. (x,y,z,t) désigne le syst. de coord. relatif à la base canonique B de R4. On cosidère la fonction définie sur  R4 par q

                         q(x,y,z,t]) = x2 + y2 + 2 z2 + 2 x y + 4 z t

1. Justifier rapidement pourquoi q est une forme quadratique.

2.Ecrire la matrice MatB(q) dans la base B .

3. Donner l'expression de la forme polaire (c-à-d la fbs) associée à q.

4. Décomposer q en somme de carrés de formes linéaires indépendantes

5. Quelle est la signature de q, son rang et son noyau.

6. Trouver une base B' = (e'1,..., e'4) (correspondant à un système de coord (x',y',z',t') dans laquelle MatB'(q) soit diagonale avec des termes diagonaux égaux à 1,-1,ou 0 .

A la question 4, sauf erreur, j'ai utilisé correctement la méthode de Gauss et obtenu:

q(x,y,z,t) = (x+y)2 + 1/4 (3z + 2t)2 - 1/4 (z-2t)2

Donc q a pour signature (2,1) et pour rang 3.
D'après le théorème du rang, le noyau est de dimension 4-3 = 1. on voit sur la matrice qu'il est engendré par (1,-1,0,0) .

A la question 6, j'ai écris:

Faisons le changement de variable

\left\lbrace\begin{matrix} x'= x + y\\ y' = 3z + 2t\\ z' = z - 2 t\\ t' = t \end{matrix}\right.

et là, je n'ai pas su comment continuer.

Merci de votre aide.

Posté par
Camélia Correcteur
re : forme quadratique 16-03-18 à 14:55

Bonjour

Je n'ai pas vérifié tes calculs. Mais il serait plus raisonnable de prendre y'=(3z+2t)/2 et z'=(z-2t)/2

Posté par
lafol Moderateur
re : forme quadratique 16-03-18 à 16:54

Bonjour
en tenant compte de la remarque de Camélia, pour avoir des 1 ou -1 ou 0 et pas d'autres nombres sur ta diagonale :
les vecteurs e' ont pour coordonnées dans la base des e' : (1,0,0,0), (0,1,0,0) etc
pour les exprimer dans la base B de départ, tu as juste à résoudre les systèmes x' = 1, y'=0, z' = 0 t' = 0 pour le premier, idem en mettant 0,1,0,0 comme second membre pour le deuxième et ainsi de suite

Posté par
carpediem
re : forme quadratique 16-03-18 à 19:10

salut

Citation :
A la question 4, sauf erreur, j'ai utilisé correctement la méthode de Gauss et obtenu:

q(x,y,z,t) = (x+y)2 + 1/4 (3z + 2t)2 - 1/4 (z-2t)2
on peut utiliser bêtement des formules ... puis on peut aussi penser ... et travailler comme au collège :

q(x, y, z, t) = x^2 + y^2 + 2z^2 + 2xy + 4zt = (x + y)^2 + 2[z^2 + 2zt + t^2 - t^2] = (x + y)^2 + 2(z + t)^2 - 2t^2

qui me semble bien plus simple ..., non ?

Posté par
scoatarin
re : forme quadratique 17-03-18 à 10:58

Merci pour vos nombreuses indications, mais je n'arrive toujours pas à comprendre la méthode et par conséquent à répondre à cette question.

En utilisant le changement de variable préconisé par camélia (que j'ai compris), et en suivant les indications de lafol, voilà ce que je trouve pour les vecteurs e':

Pour le premier vecteur e' :  t = 0, z = 0, x + y = 1

Pour le deuxième vecteur e': t =0, z = 0, x + y = 0

Pour le troisième vecteur e' : t =0, z = 1,

et je me suis arrêté là, car je ne comprends pas ce que je vais pouvoir en déduire.

A l'aide, s'il vous plaît       

Posté par
lafol Moderateur
re : forme quadratique 17-03-18 à 12:13

tu choisis des valeurs de x et y qui permettent de construire trois vecteurs linéairement indépendants qui satisfont à ces contraintes
reste à compléter avec avec un quatrième vecteur qui ne modifiera pas le calcul de q, donc qui vérifiera x'=y'=z'=0

Posté par
scoatarin
re : forme quadratique 17-03-18 à 14:25

Pour le premier vecteur e', on peut choisir x=1, y=0 ce qui donne v1= (1,0,0,0),

Pour le deuxième vecteur e', on peut choisir x=1, y=-1 ce qui donne v2= (1,-1,0,0),

Pour le troisième vecteur e', on peut choisir x=1, y=-1 et comme z=1, cela qui donne v1= (1,-1,1,0),

On remarque que ces trois vecteurs sont linéairement indépendants.

On peut compléter un quatrième vecteur v4=(0,0,0,1) qui vérifie x'=y'=z'=0.

On  obtient  une base de vecteurs B' = (e'1,..., e'4) (correspondant à un système de coord (x',y',z',t')) de la forme quadratique q.

Comment vérifier que cette base répond bien à la question 6 ?    

Posté par
Camélia Correcteur
re : forme quadratique 17-03-18 à 14:52

Mais en calculant q(v_1,v_2,v_3,v_4)

Posté par
Camélia Correcteur
re : forme quadratique 17-03-18 à 14:53

Pardon, q(v_1),q(v_2),q(V_3),q(v_4)

Posté par
scoatarin
re : forme quadratique 17-03-18 à 17:17

Camélia @ 17-03-2018 à 14:53

Pardon, q(v_1),q(v_2),q(V_3),q(v_4)


En calculant q(v1), q(v2),q(v3 ), q(v4), je n'obtient pas de matrice diagonale .

J'ai probablement fait une ou plusieurs erreurs, mais je ne vois pas ou.

A la question 2, j'ai trouvé que la base B est : Mat B (q) = \begin{pmatrix} 1 &1 &0 &0 \\ 1 &1 &0 &0 \\ 0&0 &2 &2 \\ 0 &0 &2 &0 \end{pmatrix}
et à la correction, le prof a considéré que c'était juste.
  

Posté par
carpediem
re : forme quadratique 17-03-18 à 17:25

vu la signature de q ne devrait-il pas y avoir un moins quelque part ?

Posté par
scoatarin
re : forme quadratique 17-03-18 à 17:49

carpediem @ 17-03-2018 à 17:25

vu la signature de q ne devrait-il pas y avoir un moins quelque part ?


Oui, il y a bien un signe moins mais celui-ci  apparaît seulement ici quand on écris q comme somme de carrés de formes linéaires linéairement indépendantes.

Posté par
scoatarin
re : forme quadratique 18-03-18 à 14:08

Bonjour,

Quelqu'un pourrait-il m'aider à essayer de terminer cet exercice ?

Merci.

Posté par
lafol Moderateur
re : forme quadratique 18-03-18 à 18:46

Peux-tu écrire la matrice de q dans ta nouvelle base ?

Posté par
Razes
re : forme quadratique 18-03-18 à 19:09

Bonjour,

J'utiliserais plutôt :

q(x,y,z,t)=(x+y)^2+\frac{1}{4}(3z+2t)^2-\frac{1}{4}(z-2t)^2 =(x+y)^2+\left (\frac{3z+2t}{2}\right )^2-\left (\frac{z-2t}{2}\right )^2

\left\lbrace\begin{matrix} x'= x + y\\ y' = \dfrac{3z+2t}{2}\\ z' = \dfrac{z-2t}{2}\\ t' = t \end{matrix}\right.

Posté par
lafol Moderateur
re : forme quadratique 18-03-18 à 19:12

c'est bien ce qu'on lui dit depuis 2 jours .....

Posté par
Razes
re : forme quadratique 18-03-18 à 19:31

Effectivement,  je viens de voir ce qu'avait proposé Camélia. Peut-être que scoatarin n'avait pas saisi le sens de la remarque.

Posté par
scoatarin
re : forme quadratique 18-03-18 à 20:05

Razes @ 18-03-2018 à 19:31

Effectivement,  je viens de voir ce qu'avait proposé Camélia. Peut-être que scoatarin n'avait pas saisi le sens de la remarque.


Non, j'ai bien compris le sens de la remarque mais je ne sais si mes calculs précédents sont justes ou faux  ou si je n'ai pas bien appliquer la vérification proposée par camélia car la vérification que j'ai tenté de faire n'aboutit pas. Bref, je suis dans la panade

Posté par
lafol Moderateur
re : forme quadratique 18-03-18 à 22:53

as-tu exprimé q(u) en fonction de x',y',z',t' ?
(avec u = (x,y,z,t), de coordonnées (x',y',z',t') dans la nouvelle base )

Posté par
lafol Moderateur
re : forme quadratique 18-03-18 à 23:04

il y a une erreur de base dans ton système : tu as forcé t' = t
or ça rend impossible de trouver un vecteur non nul qui satisfasse x+y = 0, (3z+2t)/2 = 1,(z-2t)/2 = 0 et t =0...
les seules choses qu'on impose pour obtenir q(u) = x'² + y'² -z'², ce qui donnera bien une matrice diagonale avec des 1, -1 ou 0 sur la diagonale, c'est x + y = x', (3z+2t)/2 = y' et (z-2t)/2 = z'

Posté par
Razes
re : forme quadratique 18-03-18 à 23:08

Voici ce que j'ai trouvé. (c'est une base orthogonale)

q(x,y,z,t)=(x+y)^2+\frac{1}{4}(3z+2t)^2-\frac{1}{4}(z-2t)^2

\left\{\begin{matrix}x'=x+y\\ y'=y\\ z'=3z+2t\\t'=z-2t\end{matrix}\right.\Leftrightarrow\left\{\begin{matrix}x=&x'-&y'\\ y=&&y'\\ z=&&&\frac{1}{4}z'+&\frac{1}{4}t'\\ t=&&&\frac{1}{8}z'-&\frac{3}{8}t'\end{matrix}\right.

v_1=\begin{pmatrix}1\\ 0\\ 0\\ 0\end{pmatrix};v_2=\begin{pmatrix}-1\\ 1\\ 0\\ 0\end{pmatrix};v_1=\begin{pmatrix}0\\ 0\\ \frac{1}{4}\\ \frac{1}{4}\end{pmatrix};v_1=\begin{pmatrix}0\\ 0\\ \frac{1}{4}\\ -\frac{1}{4}\end{pmatrix}

A toi de vérifier.

Posté par
Razes
re : forme quadratique 18-03-18 à 23:10

Rectification:
v_3=\begin{pmatrix}0\\ 0\\ \frac{1}{4}\\ \frac{1}{8}\end{pmatrix};v_4=\begin{pmatrix}0\\ 0\\ \frac{1}{4}\\ -\frac{1}{8}\end{pmatrix}

Posté par
Razes
re : forme quadratique 18-03-18 à 23:15

Décidément: v_4=\begin{pmatrix}0\\ 0\\ \frac{1}{4}\\ -\frac{3}{8}\end{pmatrix}

Posté par
Razes
re : forme quadratique 18-03-18 à 23:16

v_1,v_2,v_3,v_4 forment une base q-orthogonale

Posté par
lafol Moderateur
re : forme quadratique 18-03-18 à 23:19

on lui demandait uniquement des 1, -1 ou 0 sur la diagonale ....
pas la peine de venir répéter ce que Camélia avait dit deux jours plus tôt si c'est pour refaire l'erreur initiale ensuite ....

Posté par
scoatarin
re : forme quadratique 19-03-18 à 12:38

Merci beaucoup à tous pour votre aide intensive et le temps que vous avez consacré à me répondre.

Je pense avoir enfin compris et à titre de vérification, voilà ce que j'ai fait:

Cherchons un vecteur v2 tel que le système d'équations suivant soit vérifié :

\left\lbrace\begin{matrix} x + y = 0\\  \dfrac{3z+2t}{2} = 1\\ \dfrac{z-2t}{2} = 0\\ \end{matrix}\right.

pour x = 1 et y = -1, on a x+y = 0

et les deux autres égalités sont satisfaites si z = 1/2 et t = 1/4. d'où le vecteur v2 = (1,-1,1/2,1/4) .

On calcule q(v2) pour vérifier :

q(v2) = 1   {1,-1,0},  donc pour le vecteur v2, c'est bon !

Posté par
lafol Moderateur
re : forme quadratique 19-03-18 à 13:41

Posté par
scoatarin
re : forme quadratique 19-03-18 à 15:06

Merci encore à tous en espérant que tout est pardonné

La Fontaine - « Patience et longueur de temps font plus que force ni que rage »



Vous devez être membre accéder à ce service...

Pas encore inscrit ?

1 compte par personne, multi-compte interdit !

Ou identifiez-vous :


Rester sur la page

Inscription gratuite

Fiches en rapport

parmi 1674 fiches de maths

Désolé, votre version d'Internet Explorer est plus que périmée ! Merci de le mettre à jour ou de télécharger Firefox ou Google Chrome pour utiliser le site. Votre ordinateur vous remerciera !